如何反演斐波那契数的比奈公式-数学堆栈交换 math.stackexchange.com最新30条 2024-06-13T10:26:32Z https://math.stackexchange.com/feeds/question/4341134网址 https://creativecommons.org/licenses/by-sa/4.0/rdf https://math.stackexchange.com/q/4341134 5 如何反演斐波那契数的比奈公式 香港Bst https://math.stackexchange.com/users/261514 2021-12-24T12:24:13Z 2021-12-25T02:44:36Z <p><a href=“https://en.wikipedia.org/wiki/Fibonacci_number#计算_by_rounding“rel=”noreferrer“>根据维基百科</a>,可以将Binet的斐波那契数公式倒置:</p><p>$$F_n=\frac{\varphi^n-\psi^n}{\varpi-\psi}=\frac{\varfi^n-\psi ^n}{\sqrt 5}$$</span>其中$\varphi=\frac{1+\sqrt{5}}{2}约1.61803\,39887\ldots$和$\psi=\frac{1-\sqrt{5}}{2}=1-\varphi=-{1\over\varphi}\近似-0.61803\,39887\ldots$</span>,或者更具体地说是其截断变量:</p><p>$$F_n=\left\lfloor\frac{\varphi^n}{\sqrt 5}+\frac{1}{2}\right\rfloor$$</p><p>找到不大于实数的最大斐波那契数$F&gt的索引$n(F)$</span>;1美元</span>:</p><p>$$(F)=\left\lfloor\log_\varphi(F\sqrt5+1/2)\right\rfloor$$</p><p>由于没有给出理由,除了楼层函数是单调的之外,我想证明这一点</p>(第页)<p>因此,假设截断公式成立:$$F_n=\left\lfloor\frac{\varphi^n}{\sqrt 5}+\frac{1}{2}\right\rfloor$$</span>$$F_n=\frac{\varphi^n}{\sqrt 5}+\frac{1}{2}+E,\text{where$0\le E&lt;1$}$$</span>$$\varphi^n=\sqrt5(F_n-\frac{1}{2}-E),\text{where$0\le E&lt;1$}$$</span>$$n\log\varphi=\log(\sqrt5(F_n-\frac{1}{2}-E)),\text{where$0\le E&lt;1$}$$</span>$$n=\log_\varphi(\sqrt5(F_n-\frac{1}{2}-E)),\text{where$0\le E&lt;1$}$$</p><p>由于n是一个整数,我们可以发言:</p><p>$$n=\lfloor n\rfloor=\left\lfloor\log_\varphi(\sqrt5(F_n-\frac{1}{2}-E))\right\rfloor,\text{where$0\le E&lt;1$}$$</span>$$n=\left\lfloor\log_\varphi(\sqrt5 F_n\left(1-\frac{1+2E}{2F_n}\right)\right\rfloor,\text{where$0\le E&lt;1$}$$</span>$$n=\left\lfloor\log_\varphi(\sqrt5 F_n)+\log_\ varphi\left(1-\frac{1+2E}{2F_n}\right)\right\rfloor,\text{where$0\le E&lt;1$}$$</span></p><p>这似乎不是正确的道路</p>(第页) https://math.stackexchange.com/questions/4341134/-/434161#4341161 6 Claude Leibovic关于如何反演Binet的斐波那契数公式的回答 克劳德·莱博维奇 https://math.stackexchange.com/users/82404 2021-12-24T13:15:46Z 2021-12-25T02:44:36Z <p>我们有<span class=“math container”>$$F_n=\frac{\phi^n-(-\frac{1}{\phi})^n}{\sqrt{5}}$$</span>什么可以反转写<span class=“math-container”>$$\phi^{2n}-F_n\sqrt{5}\,\phi^n-(-1)^n=0$$</span>,它是$\phi^n$</sspan>中的二次方。所以<span class=“math container”>$$\phi^n=\frac{F_n\sqrt{5}+\sqrt{5F_n^2 \pm 4}}{2}\暗示n_\pm\log(\phi)=\log\Bigg[\frac{F_n\sqrt{5}+\sqrt{5F_n^2 \pm 4}}{2}\Bigg]$$</span>两种解决方案之一是整数<span class=“math-container”>$$\左(\开始{数组}{cccc}n(&amp;n);F_n(&amp;);n-和;氮+\\3&amp;2和amp;3000万&amp;3.209573980 \\4&amp;3&amp;3.907487979和;4.000000000 \\5&amp;5&amp;500亿&amp;5.033256487 \\6和amp;8&amp;5.987011534和;6.000000000 \\7&amp;13和;70000000000&amp;7.004918572 \\8&amp;21和;7.998115113和;8.000000000 \\9&amp;34和;90000000000&amp;2007年9月19061日\\10&amp;55岁及以上;9.999725212和;10\结束{数组}\右)$$</p><p>如果不是这样,那么这个数字就不是斐波那契。尝试<span class=“math-container”>$12345678987654321$,结果是$n_pm=78.669724$;所以,它不是斐波那契数(但我们知道最接近的一个)。$$F_{78}=8944394323791464\quad&lt;\四线组12345678987654321$$</span>$$F{79}=14472334024676221\quad&gt;\四元12345678987654321$$</span></p><p>编辑</p><p>考虑两个根之间的区别:考虑$F_n$</span>是大的,并进行一系列扩展$$n_+-n_-\sim\frac{2}{5F_n^2\log(\phi)}&lt;\压裂{1}{F_n^2}$$</p>